CARS question

This forum made possible through the generous support of SDN members, donors, and sponsors. Thank you.
D

deleted647690

Hey, I wanted to know if anyone wanted to clarify something for me from a CARS question I got wrong on the AAMC Q pack

So I chose A because the passage says, "Such firms tend to be protected from domestic or international competitors by technological advances, large-scale production, or government regulations."

This seems to fit option A, and I don't really see their reasoning for why it is wrong. They say that the author makes no reference in the passage, but he does, "...protected from domestic..competitors by....government regulations."
Yeah, I know he doesn't get too specific, but you could argue the same for why B is wrong. They never discuss such a policy as listed in B, and they are choosing this answer through outside reasoning, just as I did with A. So I don't see why A would be wrong.
Choice A seems to me like less of a stretch beyond what the author says than choice B.

Members don't see this ad.
 

Attachments

  • employers 1.PNG
    employers 1.PNG
    199.7 KB · Views: 49
  • snipping tool.PNG
    snipping tool.PNG
    174.5 KB · Views: 46
  • workers.PNG
    workers.PNG
    114.5 KB · Views: 49
Hey, I wanted to know if anyone wanted to clarify something for me from a CARS question I got wrong on the AAMC Q pack

So I chose A because the passage says, "Such firms tend to be protected from domestic or international competitors by technological advances, large-scale production, or government regulations."

This seems to fit option A, and I don't really see their reasoning for why it is wrong. They say that the author makes no reference in the passage, but he does, "...protected from domestic..competitors by....government regulations."
Yeah, I know he doesn't get too specific, but you could argue the same for why B is wrong. They never discuss such a policy as listed in B, and they are choosing this answer through outside reasoning, just as I did with A. So I don't see why A would be wrong.
Choice A seems to me like less of a stretch beyond what the author says than choice B.

The author makes no reference to regulatory policies regarding forcing the training of workers.

"Such firms tend to be protected from domestic or international competitors by technological advances, large-scale production, or government regulations."
The author is not specifically about forcing firms to train workers here at all. The author is simply saying the firms are protected in various ways. The "regulations" may or may not be regarding the training of workers but it would be too much of a stretch to just assume that.
 
The author makes no reference to regulatory policies regarding forcing the training of workers.

"Such firms tend to be protected from domestic or international competitors by technological advances, large-scale production, or government regulations."
The author is not specifically about forcing firms to train workers here at all. The author is simply saying the firms are protected in various ways. The "regulations" may or may not be regarding the training of workers but it would be too much of a stretch to just assume that.

Couldn't you say it's a stretch to assume what they did to answer B?
 
Couldn't you say it's a stretch to assume what they did to answer B?

Definitely. I remember I got this question wrong and chose the same answer you did. I thought it was a terrible question overall. I didn't encounter anything like this question on the actual mcat if that helps.
 
  • Like
Reactions: 1 user
Top